La expresión del potencial retardado causal para t<0t<0t<0 debe dar 000 pero mi cálculo produce un resultado distinto de cero. ¿Cuál es el error?

Esta pregunta se hizo anteriormente aquí en el StackExchange de Matemáticas pero usando una notación ligeramente diferente. Pero no encontré la respuesta que buscaba o más bien obtuve dos respuestas muy diferentes.

Considere la siguiente integral

I = d ω mi i ω t ω 2 C 2 k 2
que tiene polos en ω = ± C k . Cuando existen polos en la línea real, podemos sangrar el contorno para evitar los polos (o de manera equivalente, usar un i ε -prescripción).

Elijamos un contorno que se encuentre completamente en la mitad superior del complejo. ω avión. Deje que el contorno consta de dos pequeños C1 y C2, cada uno de radio ε , que pasa por encima de los polos en ω = ± C k , y el semicírculo mayor C también se encuentra en la mitad superior del plano complejo. Consulte el diagrama de las notas de la conferencia de Tong sobre electromagnetismo en la página 125, figura 50.

Con esta elección de contorno, I Tong demuestra que I = 0 para t < 0 .

Pero cuando trato de calcularlo, para diferentes partes del contorno, pieza por pieza, obtengo un resultado distinto de cero. Déjame discutir cómo. Usando el teorema del residuo de Cauchy y el lema de Jordan, da (esquemáticamente),

0 = C k ε + C k ε C k + ε + C k + ε C k ε + C k ε C k + ε + C k + ε + .

en el límite, ε 0 , la suma de las integrales 1, 3 y 5 se reduce a la integral requerida I . De este modo,

I + C k ε C k + ε + C k ε C k + ε = 0.
Por lo tanto, en el límite ε 0 ,
I = i π R mi s ( z = + C k ) + i π R mi s ( z = + C k ) = i π ( mi i C k t 2 C k ) + i π ( mi i C k t 2 C k ) = π C k pecado ( C k t ) 0

Por lo tanto, esto no concuerda con las notas de Tong que vinculé. ¿Qué está mal con mi cálculo? El resultado de Tong es correcto pero no puedo reproducirlo a mi manera. Tenga en cuenta que no estoy usando directamente el i ϵ -prescripción de polos desplazables pero utilizando la prescripción de contornos dentados.

Respuestas (2)

Su cálculo del valor principal de la integral a lo largo de la línea real es (creo) correcto. El problema es que la función de Green retardada no se define como este valor principal, sino que se define como la integral a lo largo del contorno que incluye los semicírculos .

Esta integral aparece en el contexto de la teoría cuántica de campos: hay una buena discusión sobre las diferentes formas de definir tales integrales en las notas de clase QFT de Timo Weigand , sección 1.11.3.

Considere ese diagrama que tiene allí. El propagador retardado tiene el contorno corriendo sobre ambos polos, pase lo que pase. Cuando t < 0 , debemos ejecutar la integral en el semiplano superior para aplicar el lema de Jordan para eliminar la parte semicircular del contorno. Sin embargo, cuando hacemos eso, no hay polos dentro del contorno, así que por el teorema del residuo,

C F ( t ) = F ( t ) = 0
Cuando t > 0 , debemos ejecutar la integral sobre el semiplano inferior; ambos polos están en esto, entonces ejecutas el teorema del residuo como de costumbre.

Olvidaste la contribución de los pequeños semicírculos. En realidad, creo que ahora tengo la respuesta. El comentario de Arthur Morris es correcto. Consulte también la respuesta y los comentarios math.stackexchange.com/questions/4106170/… del usuario.
@mithusengupta123 independientemente de los pequeños semicírculos, todavía no hay polos encerrados por el contorno en el t < 0 , por lo que es automáticamente cero por el teorema integral de Cauchy
Sí, pero solo si no estás hablando de integral de valor principal.